36
LSAT * PrepTest 3 Test ID: LL3003

PT03

  • Upload
    cool908

  • View
    89

  • Download
    6

Embed Size (px)

Citation preview

Page 1: PT03

LSAT*

PrepTest 3Test ID: LL3003

Page 2: PT03

A complete version of PrepTest III has been reproduced withthe permission of Law School Admission Council, Inc.

Prep Test III © 1991 Law School Admission Council, Inc.

All actual LSAT questions printed within this work are usedwith the permission of Law School Admission Council, Inc.,Box 2000, Newton, PA 18940, the copyright owner. LSACdoes not review or endorse specific test preparation or services, and inclusion of licensed LSAT questions within thiswork does not imply the review or endorsement of LSAC.

©2003 Kaplan Educational Centers

All right reserved. No part of this book may be reproduced in any form, byphotostat, microfilm, xerography, or any other means, or incorporated into any

information retrieval system, electronic or mechanical, without the writtenpermission of Kaplan Educational Centers.

Page 3: PT03

� Analytical Reasoning . . . . . . . . . . . . . . . . . . . . . . . . . . . . . . . . . SECTION I

� Logical Reasoning. . . . . . . . . . . . . . . . . . . . . . . . . . . . . . . . . . . . SECTION II

� Reading Comprehension . . . . . . . . . . . . . . . . . . . . . . . . . . . . . SECTION III

� Logical Reasoning. . . . . . . . . . . . . . . . . . . . . . . . . . . . . . . . . . . . SECTION IV

Page 4: PT03

Questions 1–7

Three couples—John and Kate, Lewis and Marie, and Nat andOlive have dinner in a restaurant together. Kate, Marie, andOlive are women; the other three are men. Each person ordersone and only one of the following kinds of entrees: porkchops, roast beef, swordfish, tilefish, veal cutlet. The six peopleorder in a manner consistent with the following conditions:

The two people in each couple do not order the samekind of entree as each other.

None of the men orders the same kind of entree asany of the other men.

Marie orders swordfish.Neither John nor Nat orders a fish entree.Olive orders roast beef.

1. Which one of the following is a complete andaccurate list of the entrees any one of which Lewiscould order?

(A) pork chops, roast beef(B) pork chops, veal cutlet(C) pork chops, swordfish, veal cutlet(D) pork chops, roast beef, tilefish, veal cutlet(E) pork chops, roast beef, swordfish, tilefish, veal

cutlet

2. Which one of the following statements could be true?

(A) John orders the same kind of entree as Mariedoes.

(B) Kate orders the same kind of entree as Natdoes.

(C) Lewis orders the same Kind of entree as Natdoes.

(D) Marie orders the same kind of entree as Olivedoes.

(E) Nat orders the same kind of entree as Olivedoes.

3. Which one of the following statements must be true?

(A) One of the men orders pork chops or vealcutlet.

(B) One of the men orders swordfish or veal cutlet.(C) Two of the women order tilefish.(D) None of the men orders a fish entree.(E) Exactly one of the women orders a fish entree.

4. If John orders veal cutlet, then which one of thefollowing statements must be true?

(A) Kate orders roast beef.(B) Kate orders swordfish.(C) Lewis orders tilefish.(D) Lewis orders veal cutlet.(E) Nat orders pork chops.

5. If none of the six people orders pork chops, thenwhich one of the following statements must be true?

(A) John orders veal cutlet.(B) Kate orders tilefish.(C) Lewis orders tilefish.(D) One of the men orders swordfish.(E) One of the women orders tilefish.

6. If Lewis orders pork chops, then which one of thefollowing is a complete and accurate list of theentrees any one of which John could order?

(A) roast beef(B) veal cutlet(C) roast beef, veal cutlet(D) roast beef, swordfish(E) pork chops, roast beef, swordfish

7. Suppose that the people in each couple both orderthe same kind of entree as each other rather thanorder different kinds of entrees. If all otherconditions remain the same, and no two womenorder the same kind of entree, then which one of thefollowing statements could be true?

(A) John orders roast beef.(B) John orders swordfish.(C) Kate orders roast beef.(D) Two of the people order pork chops.(E) Two of the people order tilefish.

GO ON TO THE NEXT PAGE.

1 -2- 11SECTION I

Time—35 minutes

24 Questions

Directions: Each group of questions in this section is based on a set of conditions. In answering some of the questions, it may beuseful to draw a rough diagram. Choose the response that most accurately and completely answers each question and blacken thecorresponding space on your answer sheet.

Page 5: PT03

Questions 8–13

There are exactly seven houses on a street. Each house isoccupied by exactly one of seven families: the Kahns, Lowes,Muirs, Newmans, Owens, Piatts, Rutans. All the houses areon the same side of the street, which runs from west to east.

The Rutans do not live in the first or the last houseon the street.

The Kahns live in the fourth house from the west endof the street.

The Muirs live next to the Kahns.The Piatts live east of both the Kahns and the Muirs

but west of the Lowes.

8. Which one of the following families could live in thehouse that is the farthest east?

(A) the Kahns(B) the Muirs(C) the Newmans(D) the Piatts(E) the Rutans

9. Which one of the following families CANNOT livenext to the Kahns?

(A) the Lowes(B) the Newmans(C) the Owens(D) the Piatts(E) the Rutans

10. If the Muirs live west of the Kahns, then the RutansCANNOT live next to both

(A) the Kahns and the Piatts(B) the Lowes and the Piatts(C) the Muirs and the Piatts(D) the Muirs and the Owens(E) the Muirs and the Newmans

11. If the Newmans live immediately west of the Kahns,which one of the following statements must be false?

(A) The Owens live next to the Newmans.(B) The Owens live next to the Rutans.(C) The Piatts live next to the Lowes.(D) The Piatts live next to the Muirs.(E) The Rutans live next to the Newmans.

12. If the Owens live east of the Muirs, which one of thefollowing statements must be true?

(A) The Kahns live east of the Muirs.(B) The Kahns live west of the Rutans.(C) The Owens live west of the Lowes.(D) The Owens live east of the Piatts.(E) The Owens live west of the Piatts.

13. If the Owens live east of the Kahns, which one of thefollowing pairs of families must live next to eachother?

(A) the Kahns and the Piatts(B) the Lowes and the Owens(C) the Muirs and the Newmans(D) the Newmans and the Rutans(E) the Owens and the Piatts

GO ON TO THE NEXT PAGE.

1 1-3-1

Page 6: PT03

Questions 14–19

At an automobile exhibition, cars are displayed on eachfloor of a three-floor building. On each floor the cars areeither all family cars or all sports cars, either all new or allused, and either all production models or all researchmodels. The following conditions apply to this exhibition:

If the exhibition includes both family cars and sportscars, then each family car is displayed on a lowernumbered floor than any sports car.

The exhibition includes no used research models.The exhibition includes no research models that are

sports cars.There are new cars on floor 1.There are used cars on floor 3.

14. If there are sports cars on exactly two floors, thenwhich one of the following statements could be true?

(A) There are research models on floor 1.(B) There are sports cars on floor 1.(C) There are family cars on floor 2.(D) There are research models on floor 2.(E) There are family cars on floor 3.

15. Which one of the following statements could be true?

(A) The exhibition includes new research modelsports cars.

(B) The exhibition includes used research modelfamily cars.

(C) The exhibition includes used research modelsports cars.

(D) There are research models on exactly one floor.(E) There are research models on all three floors.

16. Which one of the following statements must be true?

(A) There are production models on floor 1.(B) There are research models on floor 1.(C) There are production models on floor 2.(D) There are production models on floor 3.(E) There are research models on floor 3.

17. If there are research models on exactly two floors,then which one of the following statements can befalse?

(A) There are family cars on floor 1.(B) There are research models on floor 1.(C) There are new cars on floor 2.(D) There are research models on floor 2.(E) There are family cars on floor 3.

18. If all the new cars in the exhibition are researchmodels, then which one of the following statementsmust be true?

(A) All the family cars in the exhibition are new.(B) All the family cars in the exhibition are

research models.(C) All the family cars in the exhibition are used.(D) All the new cars in the exhibition are family

cars.(E) All the production models in the exhibition

are family cars.

19. If all the production models in the exhibition areused, then which one of the following statementsmust be true?

(A) There are family cars on floor 1.(B) There are new cars on floor 2.(C) There are research models on floor 2.(D) There are family cars on floor 3.(E) There are sports cars on floor 3.

GO ON TO THE NEXT PAGE.

1 -4- 11

Page 7: PT03

Questions 20–24

Planes 1, 2, 3, and 4—and no others—are available to flyin an air show.

Pilots Anna, Bob, and Cindy are all aboard planesthat are flying in the show and they are the onlyqualified pilots in the show.Copilots Dave, Ed, and Fran are all aboard planesthat are flying in the show and they are the onlyqualified copilots in the show.No plane flies in the show without a qualified pilotaboard.No one but qualified pilots and qualified copilotsflies in the show.Anna will only fly in either plane 1 or plane 4. Dave will only fly in either plane 2 or plane 3.

20. If Anna flies in plane 4 and Dave flies in plane 2,which one of the following must be true?

(A) Cindy flies in either plane 1 or plane 3.(B) If Cindy flies in plane 3, Bob flies in plane 2.(C) Bob and one other person fly in plane l.(D) If Bob is aboard plane 4, Cindy flies in plane 3.(E) If Cindy is in plane 2, Bob flies in plane 3.

21. If Bob and Anna fly on the same plane, which one ofthe following must be true?

(A) Cindy flies with Dave and Ed.(B) Cindy flies with Ed.(C) Dave flies with Cindy.(D) Dave flies with Cindy, Ed, and Fran.(E) Fran flies with Ed.

22. If Cindy and Fran are the only people in one of theplanes, which one of the following must be true?

(A) Bob flies with Anna.(B) Dave flies with Ed.(C) Dave and Ed fly with Bob.(D) Dave flies with Bob.(E) Ed flies with Anna.

23. If plane 1 is used, its crew could consist of

(A) Anna, Bob, Cindy, Fran(B) Anna, Bob, Ed, Fran(C) Bob, Cindy, Ed, Fran(D) Bob, Cindy, Dave, Ed(E) Bob, Dave, Ed, Fran

24. If as many of the pilots and copilots as possible fly inplane 4, that group will consist of

(A) exactly two people(B) exactly three people(C) exactly four people(D) exactly five people(E) three pilots and two copilots

1 1-5-1

S T O PIF YOU FINISH BEFORE TIME IS CALLED, YOU MAY CHECK YOUR WORK ON THIS SECTION ONLY.

DO NOT WORK ON ANY OTHER SECTION IN THE TEST.

Page 8: PT03

2 -6- 2

1. If you have a large amount of money in the bank,your spending power is great. If your spending poweris great, you are happy. So if you have a large amountof money in the bank, you are happy.

Which one of the following most closely parallels thereasoning in the argument above?

(A) If you have good health, you can earn a lot. Ifyou can earn a lot, you can buy an expensivehouse. So if you have good health, you canhave a comfortable life.

(B) If you drink too much alcohol, you will feelsick. If you drink too much alcohol, you willhave no money left. So if you have no moneyleft, you will feel sick.

(C) If you swim energetically, your heart rateincreases. If your heart rate increases, you areoverexcited. So if you swim energetically, youare overexcited.

(D) If you take a great deal of exercise, you arephysically fit. If you take a great deal ofexercise, you are exhausted. So if you arephysically fit, you are exhausted.

(E) If you have a large amount of money in thebank, you are confident about the future. Ifyou are optimistic by nature, you areconfident about the future. So if you have alarge amount of money in the bank, you areoptimistic by nature.

2. For a television program about astrology,investigators went into the street and found twentyvolunteers born under the sign of Gemini who werewilling to be interviewed on the program and to takea personality test. The test confirmed theinvestigators’ personal impressions that each of thevolunteers was more sociable and extroverted thanpeople are on average. This modest investigation thussupports the claim that one’s astrological birth signinfluences one’s personality.

Which one of the following, if true, indicates themost serious flaw in the method used by theinvestigators?

(A) The personality test was not administered orscored personally by the investigators.

(B) People born under astrological signs otherthan Gemini have been judged by astrologersto be much less sociable than those bornunder Gemini.

(C) The personal impressions the investigatorsfirst formed of other people have tended to beconfirmed by the investigators’ laterexperience of those people.

(D) There is not likely to be a greater proportion ofpeople born under the sign of Gemini on thestreet than in the population as a whole.

(E) People who are not sociable and extrovertedare not likely to agree to participate in suchan investigation.

GO ON TO THE NEXT PAGE.

22SECTION II

Time—35 minutes

25 Questions

Directions: The questions in this section are based on the reasoning contained in brief statements or passages. For somequestions, more than one of the choices could conceivably answer the question. However, you are to choose the best answer; thatis, the response that most accurately and completely answers the question. You should not make assumptions that are bycommonsense standards implausible, superfluous, or incompatible with the passage. After you have chosen the best answer,blacken the corresponding space on your answer sheet.

Page 9: PT03

2-7-23. In Europe, schoolchildren devote time during each

school day to calisthenics. North American schoolsrarely offer a daily calisthenics program. Tests provethat North American children are weaker, slower, andshorter-winded than European children. We mustconclude that North American children can be madephysically fit only if they participate in schoolcalisthenics on a daily basis.

Which one of the following is assumed in thepassage?

(A) All children can be made physically fit by dailycalisthenics.

(B) All children can be made equally physically fitby daily calisthenics.

(C) Superior physical fitness produces superiorhealth.

(D) School calisthenics are an indispensable factorin European children’s superior physicalfitness.

(E) North American children can learn to eat amore nutritious diet as well as to exercisedaily.

4. A work of architecture, if it is to be both inviting andfunctional for public use, must be unobtrusive,taking second place to the total environment.Modern architects, plagued by egoism, have violatedthis precept. They have let their strong personalitiestake over their work, producing buildings that arenot functional for public use.

Which one of the statements below follows logicallyfrom the statements in the passage?

(A) Unobtrusive architecture is both inviting andfunctional.

(B) Modern architects who let their strongpersonalities take over their work producebuildings that are not unobtrusive.

(C) An architect with a strong personality cannotproduce buildings that function well for thepublic.

(D) A work of architecture that takes second placeto the environment functions well for publicuse.

(E) A work of architecture cannot simultaneouslyexpress its architect’s personality and befunctional for public use.

5. Observatory director: Some say that funding themegatelescope will benefit only the astronomerswho will work with it. This dangerous point ofview, applied to the work of Maxwell, Newton,or Einstein, would have stifled their researchand deprived the world of beneficialapplications, such as the development of radio,that followed from that research.

If the statements above are put forward as anargument in favor of development of themegatelescope, which one of the following is thestrongest criticism of that argument?

(A) It appeals to the authority of experts whocannot have known all the issues involved inconstruction of the megatelescope.

(B) It does not identify those opposed todevelopment of the megatelescope.

(C) It launches a personal attack on opponents ofthe megatelescope by accusing them of havinga dangerous point of view.

(D) It does not distinguish between the economicand the intellectual senses of “benefit.”

(E) It does not show that the proposedmegatelescope research is worthy ofcomparison with that of eminent scientists inits potential for applications.

6. The Transit Authority’s proposal to increase fares by40 percent must be implemented. Admittedly, thisfare increase will impose a hardship on some bus andsubway riders. But if the fare is not increased, servicewill have to be cut severely and that would result inan unacceptably large loss of ridership.

The passage employs which one of the followingargumentative strategies?

(A) It offers evidence that the recommendedcourse of action would have no undesirableconsequences.

(B) It shows that a proponent of any alternativeposition would be forced into a contradiction.

(C) It arrives at its conclusion indirectly byproviding reasons for rejecting an alternativecourse of action.

(D) It explains why the recommended course ofaction would not be subject to the objectionsraised against the alternative.

(E) It justifies the conclusion by showing that sucha course of action has proven effective in thepast.

GO ON TO THE NEXT PAGE.

22

Page 10: PT03

2 -8- 27. Those who participate in local politics include people

who are genuinely interested in public service andpeople who are selfish opportunists. Everyone whoparticipates in local politics has an influence on thecommunity’s values.

If the statements above are true, which one of thefollowing must also be true?

(A) Some selfish opportunists have an influence onthe community’s values.

(B) Some persons who are interested in publicservice do not have an influence on thecommunity’s values.

(C) All those who have an influence on thecommunity’s values participate in localpolitics.

(D) Some of those who influence the community’svalues neither are interested in public servicenor are selfish opportunists.

(E) All those who have an influence on thecommunity’s values are either interested inpublic service or are selfish opportunists.

Questions 8–9

Although nondairy coffee lighteners made withcoconut oil contain 2 grams of saturated fat pertablespoon, or 7 times more than does whole milk, thoselighteners usually contain no cholesterol. Yet onetablespoon of such lighteners causes the consumer’s bloodcholesterol to rise to a higher level than does an identicalamount of whole milk, which contains 2 milligrams ofcholesterol per tablespoon.

8. Which one of the following, if true, contributes mostto an explanation of the apparent discrepancy notedabove?

(A) Nutritionists recommend that adults consumeas little saturated fat as possible and no morethan 250 milligrams of cholesterol a day.

(B) One gram of saturated fat in food has roughlythe same effect on blood cholesterol as 25milligrams of cholesterol in food.

(C) Light cream, a dairy product that contains 5times more cholesterol than does whole milk,is often chosen as a lightener by consumerswho normally prefer whole milk.

(D) Certain nondairy coffee lighteners madewithout coconut oil contain less saturated fatand less cholesterol than does whole milk.

(E) The lower the saturated fat content of dairyproducts, the less cholesterol they usuallycontain.

9. Manufacturers of coffee lighteners based on coconutoil claim that their products usually cause the typicalconsumer’s blood cholesterol to rise to a lower levelthan does the use of whole milk as a lightener. Whichone of the following, if true, provides the mostsupport for the manufacturers’ claim?

(A) Consumers of lighteners made with coconutoil who avoid other high-cholesterol foodsand exercise more than average tend to havelower-than-average blood cholesterol levels.

(B) Coffee is frequently consumed with pastriesand other rich desserts that themselves resultin high blood cholesterol levels.

(C) One popular nondairy coffee lightener that isnot based on coconut oil has reduced its fatcontent by 20 percent while keeping itscholesterol content at zero.

(D) Consumers typically add to their coffeesubstantially smaller quantities of coconutoil-based lighteners than of whole milk.

(E) Most consumers are convinced that wholedairy products increase blood cholesterol andthat nondairy coffee lighteners do not.

GO ON TO THE NEXT PAGE.

22

Page 11: PT03

2-9-210. People with serious financial problems are so worried

about money that they cannot be happy. Theirmisery makes everyone close to them—family,friends, colleagues—unhappy as well. Only if theirfinancial problems are solved can they and thosearound them be happy.

Which one of the following statements can beproperly inferred from the passage?

(A) Only serious problems make people unhappy.(B) People who solve their serious financial

problems will be happy.(C) People who do not have serious financial

problems will be happy.(D) If people are unhappy, they have serious

financial problems.(E) If people are happy, they do not have serious

financial problems.

11. It is often said that people should be rewarded fordoing a given job in proportion to the effort it coststhem to do it. However, a little reflection will showthat this is, in fact, a very bad idea, since it wouldmean that those people with the least skill or naturalaptitude for a particular task would be the ones giventhe most incentive to do it.

Which one of the following argumentative strategiesis used above?

(A) stating a general principle and then presentingreasons in favor of adopting it

(B) providing evidence that where the principleunder discussion has been adopted, theresults usually have been undesirable

(C) demonstrating that a consequence that hadbeen assumed to follow from the principleunder consideration need not follow from it

(D) attempting to undermine a general principleby arguing that undesirable consequenceswould follow from it

(E) showing that, in practice, the principle underconsideration could not be uniformly applied

12. Photovoltaic power plants produce electricity fromsunlight. As a result of astonishing recent technologicaladvances, the cost of producing electric power atphotovoltaic power plants, allowing for bothconstruction and operating costs, is one-tenth of whatit was 20 years ago, whereas the corresponding cost fortraditional plants, which burn fossil fuels, hasincreased. Thus, photovoltaic power plants offer a lessexpensive approach to meeting demand for electricitythan do traditional power plants.

The conclusion of the argument is properly drawn ifwhich one of the following is assumed?

(A) The cost of producing electric power attraditional plants has increased over the past20 years.

(B) Twenty years ago, traditional power plantswere producing 10 times more electric powerthan were photovoltaic plants.

(C) None of the recent technological advances inproducing electric power at photovoltaicplants can be applied to producing power attraditional plants.

(D) Twenty years ago, the cost of producingelectric power at photovoltaic plants was lessthan 10 times the cost of producing power attraditional plants.

(E) The cost of producing electric power atphotovoltaic plants is expected to decreasefurther, while the cost of producing power attraditional plants is not expected to decrease.

13. If that insect is a bee, it can only sting once. It onlydid sting once. So it is a bee.

Which one of the following exhibits a pattern ofreasoning most similar to that in the argument above?

(A) Spring is here. It has to be, because when it isspring, I cannot stop sneezing; and I justsneezed.

(B) When the sky is clear, the atmosphericpressure is high. At the moment, it is clearingup, so the atmospheric pressure is bound tobe high soon.

(C) Old and brittle paintings are always movedwith extreme care. That particular painting isnever moved with extreme care. So it mustnot be old and brittle.

(D) Only one more thunderstorm was needed toruin that roof. But the roof was still fine amonth later. There must not have been anythunderstorms over that month.

(E) To survive in the wild requires physicalstamina like Mark’s. All the same, Mark’s fearof spiders would prevent his survival.

GO ON TO THE NEXT PAGE.

22

Page 12: PT03

2 -10- 214. Pamela: Physicians training for a medical specialty

serve as resident staff physicians inhospitals.They work such long hours—up to36 consecutive hours—that fatigue impairstheir ability to make the best medical decisionsduring the final portion of their shifts.

Quincy: Thousands of physicians now practicinghave been trained according to the sameregimen, and records show they generallymade good medical decisions during theirtraining periods. Why should what has workedin the past be changed now?

Which one of the following, if true, is the mosteffective counter Pamela might make to Quincy’sargument?

(A) The basic responsibilities of resident staffphysicians in hospitals have not changedsubstantially over the past few decades.

(B) Because medical reimbursement policies nowpay for less recuperation time in hospitals,patients in hospitals are, on the average, moreseriously ill during their stays than in the past.

(C) It is important that emergency-room patientsreceive continuity of physician care, insofar aspossible, over the critical period afteradmission, generally 24 hours.

(D) The load of work on residentphysicians-in-training varies according to themedical specialty for which each is beingtrained.

(E) The training of physicians should includeobservation and recognition of the signsindicating a hospitalized patient’s progress ordecline over a period of at least 36 hours.

15. When a group of children who have been watchingtelevision programs that include acts of violence is sentto play with a group of children who have beenwatching programs that do not include acts of violence,the children who have been watching violent programscommit a much greater number of violent acts in theirplay than do the children who have been watchingnonviolent programs. Therefore, children at play canbe prevented from committing violent acts by notbeing allowed to watch violence on television.

The argument in the passage assumes which one ofthe following?

(A) Television has a harmful effect on society.(B) Parents are responsible for the acts of their

children.(C) Violent actions and passive observation of

violent actions are not related.(D) There are no other differences between the two

groups of children that might account for thedifference in violent behavior.

(E) Children who are treated violently willrespond with violence.

16. It is repeatedly claimed that the dumping of nuclearwaste poses no threat to people living nearby. If thisclaim could be made with certainty, there would beno reason for not locating sites in areas of densepopulation. But the policy of dumping nuclear wasteonly in the more sparsely populated regionsindicates, at the very least, some misgiving aboutsafety on the part of those responsible for policy.

Which one of the following, if true, would mostseriously weaken the argument?

(A) Evacuation plans in the event of an accidentcould not be guaranteed to work perfectlyexcept where the population is small.

(B) In the event of an accident, it is certain thatfewer people would be harmed in a sparselypopulated than in a densely populated area.

(C) Dumping of nuclear waste poses fewer economicand bureaucratic problems in sparselypopulated than in densely populated areas.

(D) There are dangers associated with chemicalwaste, and it, too, is dumped away from areasof dense population.

(E) Until there is no shred of doubt that nucleardumps are safe, it makes sense to situate themwhere they pose the least threat to the public.

17. A society’s infant mortality rate is an accepted indicatorof that society’s general health status. Even though insome localities in the United States the rate is higherthan in many developing countries, in the United Statesoverall the rate has been steadily declining. This declinedoes not necessarily indicate, however, that babies inthe United States are now, on the average, healthier atbirth than they were in the past.

Which one of the following reasons, if true, moststrongly supports the claim made above about theimplications of the decline?

(A) The figure for infant mortality is compiled asan overall rate and thus masks deficiencies inparticular localities.

(B) Low birth weight is a contributing factor inmore than half of the infant deaths in theUnited States.

(C) The United States has been developing and hasachieved extremely sophisticated technologyfor saving premature and low-birth-weightbabies, most of whom require extendedhospital stays.

(D) In eleven states of the United States, the infantmortality rate declined last year.

(E) Babies who do not receive adequate attentionfrom a caregiver fail to thrive and so they gainweight slowly.

GO ON TO THE NEXT PAGE.

22

Page 13: PT03

2-11-2Questions 18–19

Like a number of other articles, Ian Raghnall’s articlerelied on a recent survey in which over half the couplesapplying for divorces listed “money” as a major problem intheir marriages. Raghnall’s conclusion from the survey datais that financial problems are the major problem inmarriages and an important factor contributing to the highdivorce rate. Yet couples often express other types of maritalfrustrations in financial terms. Despite appearances, thesurvey data do not establish that financial problems are themajor problem in contemporary marriages.

18. Which one of the following sentences best expressesthe main point of the passage?

(A) Financial problems are not an importantfactor contributing to the divorce rate.

(B) Marital problems are more easily solved bymarriage counselors than by married coupleson their own.

(C) The conclusion drawn in Raghnall’s article isinadequately justified.

(D) Over half the couples applying for divorceslisted money as a major problem in theirmarriages.

(E) Many articles wrongly claim that financialproblems are the major factor contributing tothe divorce rate.

19. In the passage, the author does which one of thefollowing?

(A) undermines a conclusion drawn fromstatistical data by offering a specificcounterexample

(B) undermines a conclusion drawn fromstatistical data by offering an alternativeexplanation for some of the data

(C) undermines a conclusion drawn fromstatistical data by showing that one cannotprove the presence of an emotion by usingstatistical methods

(D) undermines a conclusion drawn fromstatistical data by criticizing the survey forwhich the data was gathered

(E) undermines a conclusion by showing thatcouples cannot accurately describe their ownproblems

20. In Brazil, side-by-side comparisons of Africanizedhoneybees and the native honeybees have shown thatthe Africanized bees are far superior honeyproducers. Therefore, there is no reason to fear thatdomestic commercial honey production will declinein the United States if local honeybees are displacedby Africanized honeybees.

Each of the following, if true, would weaken theargument EXCEPT:

(A) The honeybees native to Brazil are not of thesame variety as those most frequently used inthe commercial beekeeping industry in theUnited States.

(B) Commercial honey production is far morecomplicated and expensive with Africanizedhoneybees than it is with the more docilehoneybees common in the United States.

(C) If Africanized honeybees replace localhoneybees, certain types of ornamental treeswill be less effectively pollinated.

(D) In the United States a significant proportion ofthe commercial honey supply comes fromhobby beekeepers, many of whom are likelyto abandon beekeeping with the influx ofAfricanized bees.

(E) The area of Brazil where the comparativestudy was done is far better suited to theforaging habits of the Africanized honeybeesthan are most areas of the United States.

21. The public is well aware that high blood cholesterollevels raise the risk of stroke caused by blood clots.But a recent report concludes that people with lowblood cholesterol levels are at increased risk of theother lethal type of stroke—cerebral hemorrhage,caused when a brain artery bursts. The reportsuggests that because blood cholesterol plays a vitalrole in maintaining cell membranes, low bloodcholesterol weakens artery walls, making them proneto rupture. The conclusion thus supports along-standing contention by Japanese researchersthat Western diets better protect against cerebralhemorrhage than do non-Western diets.

The argument is based on which one of the followingassumptions?

(A) Western diets are healthier than non-Westerndiets.

(B) Western diets result in higher bloodcholesterol levels than do non-Western diets.

(C) High blood cholesterol levels preclude theweakening of artery walls.

(D) Cerebral hemorrhages are more dangerousthan strokes caused by blood clots.

(E) People who have low blood pressure are atincreased risk of cerebral hemorrhage.

GO ON TO THE NEXT PAGE.

22

Page 14: PT03

2 -12- 222. Public reports by national commissions, governors’

conferences, and leadership groups have stressed thegreat need for better understanding of internationalaffairs by the citizenry. If the country is to remain aleading nation in an era of internationalcompetitiveness, the need is undeniable. If there issuch a need for the citizenry to have a betterunderstanding of international affairs, then all of ournew teachers must be prepared to teach their subjectmatter with an international orientation.

If all of the statements in the passage are true, whichone of the following must also be true?

(A) If the country is to remain a leading nation in anera of international competitiveness, then newteachers must be prepared to teach their subjectmatter with an international orientation.

(B) If new teachers are prepared to teach theirsubject matter with an internationalorientation, then the country will remain aleading nation in an era of internationalcompetitiveness.

(C) If there is better understanding ofinternational affairs by the citizenry, then thecountry will remain a leading nation in an eraof international competitiveness.

(D) If the country is to remain a leading nation inan era of international competitiveness, thenthere is no need for the citizenry to have abetter understanding of international affairs.

(E) Public reports from various groups andcommissions have stressed the need for amore international orientation in theeducation of teachers.

23. “DNA fingerprinting” is a recently-introducedbiochemical procedure that uses a pattern derivedfrom a person’s genetic material to match a suspect’sgenetic material against that of a specimen from acrime scene. Proponents have claimed astronomicallyhigh odds against obtaining a match by chance alone.These odds are based on an assumption that there isindependence between the different characteristicsrepresented by a single pattern.

Which one of the following, if true, casts the mostdoubt on the claim of the proponents of DNAfingerprinting?

(A) The large amount of genetic material thatpeople share with all other people and withother animals is not included in the DNAfingerprinting procedure.

(B) There is a generally accepted theoretical basisfor interpreting the patterns produced by theprocedure.

(C) In the whole population there are variousdifferent subgroups, within each of whichcertain sets of genetic characteristics are shared.

(D) The skill required of laboratory techniciansperforming the DNA fingerprintingprocedure is not extraordinary.

(E) In the investigation of certain genetic diseases,the techniques used in DNA fingerprintinghave traced the transmission of the diseasesamong the living members of very largefamilies.

GO ON TO THE NEXT PAGE.

22

Page 15: PT03

2-13-224. Anthropologists assert that cultures advance only

when independence replaces dependence—that is,only when imposition by outsiders is replaced byinitiative from within. In other words, the natives ofa culture are the only ones who can move thatculture forward. Non-natives may provide valuableadvice, but any imposition of their views threatensindependence and thus progress. If one looks atindividual schools as separate cultures, therefore, thekey to educational progress is obvious: ____ .

Which one of the following best completes thepassage?

(A) individual schools must be independent ofoutside imposition

(B) some schools require more independence thanothers, depending on the initiative of theirstaffs and students

(C) school system officials must tailor theirinitiatives for change to each individualschool in the system

(D) outsiders must be prevented fromparticipation in schools’ efforts to advance

(E) the more independent a school is, the moreeducational progress it will make

25. The public in the United States has in the past beenconditioned to support a substantial defense budget bythe threat of confrontation with the Eastern bloc. Nowthat that threat is dissolving, along with the Easternbloc itself, it is doubtful whether the public can bepersuaded to support an adequate defense budget.

Which one of the following indicates a weakness inthe position expressed above?

(A) It presupposes that public opinion can bemanipulated indefinitely, without the public’sbecoming aware of that manipulation.

(B) It refers to past and present events that do nothave a causal connection with public supportof the budget.

(C) It assumes as fact what it seeks to establish byreasoning.

(D) It fails to give any reason for the judgment itreaches.

(E) It hinges on the term “adequate,” the precisemeaning of which requires reevaluation in thenew context.

22

S T O PIF YOU FINISH BEFORE TIME IS CALLED, YOU MAY CHECK YOUR WORK ON THIS SECTION ONLY.

DO NOT WORK ON ANY OTHER SECTION IN THE TEST.

Page 16: PT03

33 -14-

Until recently many astronomers believed thatasteroids travel about the solar systemunaccompanied by satellites. These astronomersassumed this because they considered asteroid-satellite systems inherently unstable. Theoreticianscould have told them otherwise: even minusculebodies in the solar system can theoretically havesatellites, as long as everything is in proper scale. If abowling ball were orbiting about the Sun in theasteroid belt, it could have a pebble orbiting it as faraway as a few hundred radii (or about 50 meters)without losing the pebble to the Sun’s gravitationalpull.

Observations now suggest that asteroid satellitesmay exist not only in theory but also in reality.Several astronomers have noticed, while watchingasteroids pass briefly in front of stars, that somethingbesides the known asteroid sometimes blocks out thestar as well. Is that something a satellite?

The most convincing such report concerns theasteroid Herculina, which was due to pass in front ofa star in 1978. Astronomers waiting for the predictedevent found not just one occultation, or eclipse, of thestar, but two distinct drops in brightness. One wasthe predicted occultation, exactly on time. The other,lasting about five seconds, preceded the predictedevent by about two minutes. The presence of asecondary body near Herculina thus seemed stronglyindicated. To cause the secondary occultation, anunseen satellite would have to be about 45 kilometersin diameter, a quarter of the size of Herculina, and ata distance of 990 kilometers from the asteroid at thetime. These values are within theoretical bounds, andsuch an asteroid-satellite pair could be stable.

With the Herculina event, apparent secondaryoccultations became “respectable”—and morecommonly reported. In fact, so common did reportsof secondary events become that they are now simplytoo numerous for all of them to be accurate. Even ifevery asteroid has as many satellites as can be fittedaround it without an undue number of collisions, onlyone in every hundred primary occultations would beaccompanied by a secondary event (one in everythousand if asteroidal satellite systems resembledthose of the planets).

Yet even astronomers who find the case forasteroid satellites unconvincing at present say theywould change their minds if a photoelectric recordwere made of a well-behaved secondary event. By“well-behaved” they mean that during occultationthe observed brightness must drop sharply as the star

winks out and must rise sharply as it reappears frombehind the obstructing object, but the brightnessduring the secondary occultation must drop to that ofthe asteroid, no higher and no lower. This wouldmake it extremely unlikely that an airplane or aglitch in the instruments was masquerading as anocculting body.

1. Which one of the following best expresses the mainidea of the passage?

(A) The observation of Herculina represented thecrucial event that astronomical observers andtheoreticians had been waiting for to establisha convincing case for the stability of asteroidsatellite systems.

(B) Although astronomers long believed thatobservation supports the existence of stableasteroid-satellite systems, numerous recentreports have increased skepticism on thisissue in astronomy.

(C) Theoreticians’ views on the stability of asteroidsatellite systems may be revised in the light ofreports like those about Herculina.

(D) Astronomers continue to consider itrespectable to doubt the stability ofasteroid-satellite systems, but new theoreticaldevelopments may change their views.

(E) The Herculina event suggests that theoreticians’views about asteroid-satellite systems may becorrect, and astronomers agree about the kindof evidence needed to clearly resolve the issue.

2. Which one of the following is mentioned in the passageas providing evidence that Herculina has a satellite?

(A) the diameter of a body directly observed nearHerculina

(B) the distance between Herculina and the planetnearest to it

(C) the shortest possible time in which satellites ofHerculina, if any, could complete a single orbit

(D) the occultation that occurred shortly beforethe predicted occultation by Herculina

(E) the precise extent to which observed brightnessdropped during the occultation by Herculina

GO ON TO THE NEXT PAGE.

33 3SECTION III

Time—35 minutes

28 Questions

Directions: Each passage in this section is followed by a group of questions to be answered on the basis of what is stated orimplied in the passage. For some of the questions, more than one of the choices could conceivably answer the question. However,you are to choose the best answer; that is, the response that most accurately and completely answers the question, and blackenthe corresponding space on your answer sheet.

(5)

(10)

(15)

(20)

(25)

(30)

(35)

(40)

(45)

(50)

(55)

Page 17: PT03

3-15-33. According to the passage, the attitude of astronomers

toward asteroid satellites since the Herculina eventcan best be described as

(A) open-mindedness combined with a concern forrigorous standards of proof

(B) contempt for and impatience with the positionheld by theoreticians

(C) bemusement at a chaotic mix of theory,inadequate or spurious data, and calls forscientific rigor

(D) hardheaded skepticism, implying rejection of alldata not recorded automatically by state-of-the-art instruments

(E) admiration for the methodical process bywhich science progresses from initialhypothesis to incontrovertible proof

4. The author implies that which one of the followingwas true prior to reports of the Herculina event?

(A) Since no good theoretical model existed, allclaims that reports of secondary occultationswere common were disputed.

(B) Some of the reported observations ofsecondary occultations were actuallyobservations of collisions of satellites withone another.

(C) If there were observations of phenomenaexactly like the phenomena now labeledsecondary occultations, astronomers were lesslikely then to have reported suchobservations.

(D) The prevailing standards concerning what toclassify as a well-behaved secondary eventwere less stringent than they are now.

(E) Astronomers were eager to publish theirobservations of occultations of stars bysatellites of asteroids.

5. The information presented in the passage implieswhich one of the following about the frequency ofreports of secondary occultations after the Herculinaevent?

(A) The percentage of reports of primaryoccultations that also included reports ofsecondary occultations increased tenfoldcompared to the time before the Herculinaevent.

(B) Primary occultations by asteroids werereported to have been accompanied bysecondary occultations in about one out ofevery thousand cases.

(C) The absolute number of reports of secondaryoccultations increased tenfold compared tothe time before the Herculina event.

(D) Primary occultations by asteroids werereported to have been accompanied bysecondary occultations in more than one outof every hundred cases.

(E) In more than one out of every hundred cases,primary occultations were reported to havebeen accompanied by more than onesecondary occultation.

6. The primary purpose of the passage is to

(A) cast doubt on existing reports of secondaryoccultations of stars

(B) describe experimental efforts by astronomers toseparate theoretically believable observations ofsatellites of asteroids from spurious ones

(C) review the development of ideas amongastronomers about whether or not satellitesof asteroids exist

(D) bring a theoretician’s perspective to bear on anincomplete discussion of satellites of asteroids

(E) illustrate the limits of reasonable speculationconcerning the occultation of stars

7. The passage suggests that which one of the followingwould most help to resolve the question of whetherasteroids have satellites?

(A) a review of pre-1978 reports of secondaryoccultations

(B) an improved theoretical model of stablesatellite systems

(C) a photoelectric record of a well-behavedsecondary occultation

(D) a more stringent definition of what constitutesa well-behaved secondary occultation

(E) a powerful telescope that would permit acomparison of ground-based observationswith those made from airplanes

GO ON TO THE NEXT PAGE.

33 3

Page 18: PT03

33 -16-

Historians attempting to explain how scientificwork was done in the laboratory of the seventeenth-century chemist and natural philosopher RobertBoyle must address a fundamental discrepancybetween how such experimentation was actuallyperformed and the seventeenth-century rhetoricdescribing it. Leaders of the new Royal Society ofLondon in the 1660s insisted that authentic sciencedepended upon actual experiments performed,observed, and recorded by the scientists themselves.Rejecting the traditional contempt for manualoperations, these scientists, all members of theEnglish upper class, were not to think themselvesdemeaned by the mucking about with chemicals,furnaces, and pumps; rather, the willingness of eachof them to become, as Boyle himself said, a mere“drudge” and “under-builder” in the search forGod’s truth in nature was taken as a sign of theirnobility and Christian piety.

This rhetoric has been so effective that onemodern historian assures us that Boyle himselfactually performed all of the thousand or moreexperiments he reported. In fact, due to pooreyesight, fragile health, and frequent absences fromhis laboratory, Boyle turned over much of the laborof obtaining and recording experimental results topaid technicians, although published accounts of theexperiments rarely, if ever, acknowledged thetechnicians’ contributions. Nor was Boyle unique inrelying on technicians without publicly creditingtheir work.

Why were the contributions of these techniciansnot recognized by their employers? One reason is thehistorical tendency, which has persisted into thetwentieth century, to view scientific discovery asresulting from momentary flashes of individualinsight rather than from extended periods ofcooperative work by individuals with varying levels ofknowledge and skill. Moreover, despite the clamor ofseventeenth-century scientific rhetoric commending ahands-on approach, science was still overwhelminglyan activity of the English upper class, and thetraditional contempt that genteel society maintainedfor manual labor was pervasive and deeply rooted.Finally, all of Boyle’s technicians were “servants,”which in seventeenth-century usage meant anyonewho worked for pay. To seventeenth-centurysensibilities, the wage relationship was charged withpolitical significance. Servants, meaning wageearners, were excluded from the franchise becausethey were perceived as ultimately dependent on theirwages and thus controlled by the will of theiremployers. Technicians remained invisible in thepolitical economy of science for the same reasonsthat underlay servants’ general political exclusion.The technicians’ contributions, their observationsand judgment, if acknowledged, would not have beenperceived in the larger scientific community asobjective because the technicians were dependent onthe wages paid to them by their employers. Servantsmight have made the apparatus work, but theircontributions to the making of scientific knowledgewere largely—and conveniently—ignored by theiremployers.

8. Which one of the following best summarizes themain idea of the passage?

(A) Seventeenth-century scientific experimentationwould have been impossible without the workof paid laboratory technicians.

(B) Seventeenth-century social conventionsprohibited upper-class laboratory workersfrom taking public credit for their work.

(C) Seventeenth-century views of scientific discoverycombined with social class distinctions toensure that laboratory technicians’ scientificwork was never publicly acknowledged.

(D) Seventeenth-century scientists were far moredependent on their laboratory techniciansthan are scientists today, yet far less willing toacknowledge technicians’ scientificcontributions.

(E) Seventeenth-century scientists liberatedthemselves from the stigma attached tomanual labor by relying heavily on the workof laboratory technicians.

9. It can be inferred from the passage that the“seventeenth-century rhetoric” mentioned in line 6would have more accurately described theexperimentation performed in Boyle’s laboratory ifwhich one of the following were true?

(A) Unlike many seventeenth-century scientists,Boyle recognized that most scientificdiscoveries resulted from the cooperativeefforts of many individuals.

(B) Unlike many seventeenth-century scientists,Boyle maintained a deeply rooted andpervasive contempt for manual labor.

(C) Unlike many seventeenth-century scientists,Boyle was a member of the Royal Society ofLondon.

(D) Boyle generously acknowledged thecontribution of the technicians who workedin his laboratory.

(E) Boyle himself performed the actual labor ofobtaining and recording experimental results.

10. According to the author, servants in seventeenth-century England were excluded from the franchisebecause of the belief that

(A) their interests were adequately represented bytheir employers

(B) their education was inadequate to makeinformed political decisions

(C) the independence of their political judgmentwould be compromised by their economicdependence on their employers

(D) their participation in the elections would be apolarizing influence on the political process

(E) the manual labor that they performed did notconstitute a contribution to the society thatwas sufficient to justify their participation inelections

GO ON TO THE NEXT PAGE.

33 3

(5)

(10)

(15)

(20)

(25)

(30)

(35)

(40)

(45)

(50)

(55)

(60)

Page 19: PT03

3-17-311. According to the author, the Royal Society of London

insisted that scientists abandon the

(A) belief that the primary purpose of scientificdiscovery was to reveal the divine truth thatcould be found in nature

(B) view that scientific knowledge results largelyfrom the insights of a few brilliant individualsrather than from the cooperative efforts ofmany workers

(C) seventeenth-century belief that servantsshould be denied the right to vote becausethey were dependent on wages paid to themby their employers

(D) traditional disdain for manual labor that wasmaintained by most members of the Englishupper class during the seventeenth century

(E) idea that the search for scientific truth was asign of piety

12. The author implies that which one of the followingbeliefs was held in both the seventeenth and thetwentieth centuries?

(A) Individual insights rather than cooperativeendeavors produce most scientific discoveries.

(B) How science is practiced is significantlyinfluenced by the political beliefs andassumptions of scientists.

(C) Scientific research undertaken for pay cannotbe considered objective.

(D) Scientific discovery can reveal divine truth innature.

(E) Scientific discovery often relies on theunacknowledged contributions of laboratorytechnicians.

13. Which one of the following best describes theorganization of the last paragraph?

(A) Several alternative answers are presented to aquestion posed in the previous paragraph,and the last is adopted as the most plausible.

(B) A question regarding the cause of thephenomenon described in the previousparagraph is posed, two possible explanationsare rejected, and evidence is provided insupport of a third.

(C) A question regarding the phenomenondescribed in the previous paragraph is posed,and several incompatible views are presented.

(D) A question regarding the cause of thephenomenon described in the previousparagraph is posed, and several contributingfactors are then discussed.

(E) Several possible answers to a question areevaluated in light of recent discoveries citedearlier in the passage.

14. The author’s discussion of the political significanceof the “wage relationship” (line 48) serves to

(A) place the failure of seventeenth-centuryscientists to acknowledge the contributions oftheir technicians in the larger context ofrelations between workers and theiremployers in seventeenth-century England

(B) provide evidence in support of the author’smore general thesis regarding the relationshipof scientific discovery to the economicconditions of societies in which it takes place

(C) provide evidence in support of the author’sexplanation of why scientists inseventeenth-century England were reluctant torely on their technicians for the performance ofanything but the most menial tasks

(D) illustrate political and economic changes in thesociety of seventeenth-century England thathad a profound impact on how scientificresearch was conducted

(E) undermine the view that scientific discoveryresults from individual enterprise rather thanfrom the collective endeavor of many workers

15. It can be inferred from the passage that “the clamorof seventeenth-century scientific rhetoric”(lines 39–40) refers to

(A) the claim that scientific discovery resultslargely from the insights of brilliantindividuals working alone

(B) ridicule of scientists who were members of theEnglish upper class and who were thought todemean themselves by engaging in themanual labor required by their experiments

(C) criticism of scientists who publiclyacknowledged the contributions of theirtechnicians

(D) assertions by members of the Royal Society ofLondon that scientists themselves should beresponsible for obtaining and recordingexperimental results

(E) the claim by Boyle and his colleagues that theprimary reason for scientific research is todiscover evidence of divine truth in thenatural world

GO ON TO THE NEXT PAGE.

33 3

Page 20: PT03

33 -18-

One type of violation of the antitrust laws is theabuse of monopoly power. Monopoly power is theability of a firm to raise its prices above thecompetitive level—that is, above the level that wouldexist naturally if several firms had tocompete—without driving away so many customersas to make the price increase unprofitable. In order toshow that a firm has abused monopoly power, andthereby violated the antitrust laws, two essentialfacts must be established. First, a firm must beshown to possess monopoly power, and second, thatpower must have been used to exclude competition inthe monopolized market or related markets.

The price a firm may charge for its product isconstrained by the availability of close substitutes forthe product. If a firm attempts to charge a higherprice—a supracompetitive price—customers willturn to other firms able to supply substitute productsat competitive prices. If a firm provides a largepercentage of the products actually or potentiallyavailable, however, customers may find it difficult tobuy from alternative suppliers. Consequently, a firmwith a large share of the relevant market ofsubstitutable products may be able to raise its pricewithout losing many customers. For this reasoncourts often use market share as a rough indicator ofmonopoly power.

Supracompetitive prices are associated with a lossof consumers’ welfare because such prices force someconsumers to buy a less attractive mix of productsthan they would ordinarily buy. Supracompetitiveprices, however, do not themselves constitute anabuse of monopoly power. Antitrust laws do notattempt to counter the mere existence of monopolypower, or even the use of monopoly power to extractextraordinarily high profits. For example, a firmenjoying economies of scale—that is, low unitproduction costs due to high volume—does notviolate the antitrust laws when it obtains a largemarket share by charging prices that are profitablebut so low that its smaller rivals cannot survive. Ifthe antitrust laws posed disincentives to the existenceand growth of such firms, the laws could impairconsumers’ welfare. Even if the firm, upon acquiringmonopoly power, chose to raise prices in order toincrease profits, it would not be in violation of theantitrust laws.

The antitrust prohibitions focus instead on abusesof monopoly power that exclude competition in themonopolized market or involve leverage—the use ofpower in one market to reduce competition inanother. One such forbidden practice is a tyingarrangement, in which a monopolist conditions thesale of a product in one market on the buyer’spurchase of another product in a different market.For example, a firm enjoying a monopoly in thecommunications systems market might not sell itsproducts to a customer unless that customer alsobuys its computer systems, which are competing withother firms’ computer systems.

The focus on the abuse of monopoly power,rather than on monopoly itself, follows from theprimary purpose of the antitrust laws: to promoteconsumers’ welfare through assurance of the qualityand quantity of products available to consumers.

16. Which one of the following distinctions betweenmonopoly power and the abuse of monopoly powerwould the author say underlies the antitrust lawsdiscussed in the passage?

(A) Monopoly power is assessed in terms ofmarket share, whereas abuse of monopolypower is assessed in terms of market control.

(B) Monopoly power is easy to demonstrate,whereas abuse of monopoly power is difficultto demonstrate.

(C) Monopoly power involves only one market,whereas abuse of monopoly power involves atleast two or more related markets.

(D) Monopoly power is the ability to chargesupracompetitive prices, whereas abuse ofmonopoly power is the use of that ability.

(E) Monopoly power does not necessarily hurtconsumer welfare, whereas abuse ofmonopoly power does.

17. Would the use of leverage meet the criteria for abuseof monopoly power outlined in the first paragraph?

(A) No, because leverage involves anonmonopolized market.

(B) No, unless the leverage involves a tyingarrangement.

(C) Yes, because leverage is a characteristic ofmonopoly power.

(D) Yes, unless the firm using leverage is chargingcompetitive prices.

(E) Yes, because leverage is used to eliminatecompetition in a related market.

GO ON TO THE NEXT PAGE.

33 3

(5)

(10)

(15)

(20)

(25)

(30)

(35)

(40)

(45)

(50)

(55)

(60)

(65)

Page 21: PT03

3-19-318. What is the main purpose of the third paragraph

(lines 28–47)?

(A) to distinguish between supracompetitive pricesand supracompetitive profits

(B) to describe the positive uses of monopoly power(C) to introduce the concept of economies of scale(D) to distinguish what is not covered by the

antitrust laws under discussion from what iscovered

(E) to remind the reader of the issue of consumers’welfare

19. Given only the information in the passage, withwhich one of the following statements aboutcompetition would those responsible for the antitrustlaws most likely agree?

(A) Competition is essential to consumers’ welfare.(B) There are acceptable and unacceptable ways

for firms to reduce their competition.(C) The preservation of competition is the

principal aim of the antitrust laws.(D) Supracompetitive prices lead to reductions in

competition.(E) Competition is necessary to ensure

high-quality products at low prices.

20. Which one of the following sentences would bestcomplete the last paragraph of the passage?

(A) By limiting consumers’ choices, abuse ofmonopoly power reduces consumers’ welfare,but monopoly alone can sometimes actuallyoperate in the consumers’ best interests.

(B) What is needed now is a set of related laws todeal with the negative impacts that monopolyitself has on consumers’ ability to purchaseproducts at reasonable cost.

(C) Over time, the antitrust laws have been veryeffective in ensuring competition and,consequently, consumers’ welfare in thevolatile communications and computersystems industries.

(D) By controlling supracompetitive prices andcorresponding supracompetitive profits, theantitrust laws have, indeed, gone a long waytoward meeting that objective.

(E) As noted above, the necessary restraints onmonopoly itself have been left to the market,where competitive prices and economies ofscale are rewarded through increased marketshare.

GO ON TO THE NEXT PAGE.

33 3

Page 22: PT03

33 -20-

Amsden has divided Navajo weaving into fourdistinct styles. He argues that three of them can beidentified by the type of design used to formhorizontal bands: colored stripes, zigzags, ordiamonds. The fourth, or bordered, style he identifiesby a distinct border surrounding centrally placed,dominating figures.

Amsden believes that the diamond style appearedafter 1869 when, under Anglo influence andencouragement, the blanket became a rug withlarger designs and bolder lines. The bordered styleappeared about 1890, and, Amsden argues, it reflectsthe greatest number of Anglo influences on the newlyemerging rug business. The Anglo desire thatanything with graphic designs have a top, bottom,and border is a cultural preference that the Navajoabhorred, as evidenced, he suggests, by the fact thatin early bordered specimens strips of colorunexpectedly break through the enclosing pattern.

Amsden argues that the bordered rug represents aradical break with previous styles. He asserts that theborder changed the artistic problem facing weavers:a blank area suggests the use of isolated figures,while traditional, banded Navajo designs werecontinuous and did not use isolated figures. The oldpatterns alternated horizontal decorative zones in aregular order.

Amsden’s view raises several questions. First,what is involved in altering artistic styles? Somestudies suggest that artisans’ motor habits andthought processes must be revised when a stylechanges precipitously. In the evolution of Navajoweaving, however, no radical revisions in the wayarticles are produced need be assumed. After all, allweaving subordinates design to the physicallimitations created by the process of weaving, whichincludes creating an edge or border. The habitsrequired to make decorative borders are, therefore,latent and easily brought to the surface.

Second, is the relationship between the bandedand bordered styles as simple as Amsden suggests?He assumes that a break in style is a break inpsychology. But if style results from constant questsfor invention, such stylistic breaks are inevitable.When a style has exhausted the possibilities inherentin its principles, artists cast about for new, but notnecessarily alien, principles. Navajo weaving mayhave reached this turning point prior to 1890.

Third, is there really a significant stylistic gap?Two other styles lie between the banded styles andthe bordered style. They suggest that disintegrationof the bands may have altered visual and motorhabits and prepared the way for a border filled withseparate units. In the Chief White Antelope blanket,dated prior to 1865, ten years before the first Anglotrading post on the Navajo reservation, whole andpartial diamonds interrupt the flowing design andbecome separate forms. Parts of diamonds arrangedvertically at each side may be seen to anticipate theborder.

21. The author’s central thesis is that

(A) the Navajo rejected the stylistic influences ofAnglo culture

(B) Navajo weaving cannot be classified byAmsden’s categories

(C) the Navajo changed their style of weavingbecause they sought the challenge of newartistic problems

(D) original motor habits and thought processeslimit the extent to which a style can be revised

(E) the causal factors leading to the emergence ofthe bordered style are not as clear-cut asAmsden suggests

22. It can be inferred from the passage that Amsdenviews the use of “strips of color” (line 18) in the earlybordered style as

(A) a sign of resistance to a change in style(B) an echo of the diamond style(C) a feature derived from Anglo culture(D) an attempt to disintegrate the rigid form of the

banded style(E) a means of differentiating the top of the

weaving from the bottom

23. The author’s view of Navajo weaving suggests whichone of the following?

(A) The appearance of the first trading post on theNavajo reservation coincided with theappearance of the diamond style.

(B) Traces of thought processes and motor habitsof one culture can generally be found in theart of another culture occupying the sameperiod and region.

(C) The bordered style may have developedgradually from the banded style as a result ofNavajo experiments with design.

(D) The influence of Anglo culture was not theonly non-Native American influence onNavajo weaving.

(E) Horizontal and vertical rows of diamondforms were transformed by the Navajos intosolid lines to create the bordered style.

24. According to the passage, Navajo weavings madeprior to 1890 typically were characterized by all ofthe following EXCEPT

(A) repetition of forms(B) overall patterns(C) horizontal bands(D) isolated figures(E) use of color

GO ON TO THE NEXT PAGE.

33 3

(5)

(10)

(15)

(20)

(25)

(30)

(35)

(40)

(45)

(50)

(55)

(60)

Page 23: PT03

3-21-325. The author would most probably agree with which

one of the following conclusions about the stylisticdevelopment of Navajo weaving?

(A) The styles of Navajo weaving changed inresponse to changes in Navajo motor habitsand thought processes.

(B) The zigzag style was the result of stylisticinfluences from Anglo culture.

(C) Navajo weaving used isolated figures in thebeginning, but combined naturalistic andabstract designs in later styles.

(D) Navajo weaving changed gradually from a stylein which the entire surface was covered byhorizontal bands to one in which centralfigures dominated the surface.

(E) The styles of Navajo weaving always containedsome type of isolated figure.

26. The author suggests that Amsden’s claim that bordersin Navajo weaving were inspired by Anglo culturecould be

(A) conceived as a response to imaginedcorrespondences between Anglo and Navajoart

(B) biased by Amsden’s feelings about Angloculture

(C) a result of Amsden’s failing to take intoaccount certain aspects of Navajo weaving

(D) based on a limited number of specimens of thestyles of Navajo weaving

(E) based on a confusion between the stylisticfeatures of the zigzag and diamond styles

27. The author most probably mentions the Chief WhiteAntelope blanket in order to

(A) establish the direct influence of Anglo cultureon the bordered style

(B) cast doubts on the claim that the borderedstyle arose primarily from Anglo influence

(C) cite an example of a blanket with a centraldesign and no border

(D) suggest that the Anglo influence producedsignificant changes in the two earliest styles ofNavajo weaving

(E) illustrate how the Navajo had exhausted thestylistic possibilities of the diamond style

28. The passage is primarily concerned with

(A) comparing and contrasting different styles(B) questioning a view of how a style came into

being(C) proposing alternate methods of investigating

the evolution of styles(D) discussing the influence of one culture on

another(E) analyzing the effect of the interaction between

two different cultures

33 3

S T O PIF YOU FINISH BEFORE TIME IS CALLED, YOU MAY CHECK YOUR WORK ON THIS SECTION ONLY.

DO NOT WORK ON ANY OTHER SECTION IN THE TEST.

Page 24: PT03

44 -22-

1. The translator of poetry must realize thatword-for-word equivalents do not exist acrosslanguages, any more than piano sounds exist in theviolin. The violin can, however, play recognizably thesame music as the piano, but only if the violinist isguided by the nature and possibilities of the violin aswell as by the original composition.

As applied to the act of translating poetry from onelanguage into another, the analogy above can best beunderstood as saying that

(A) poetry cannot be effectively translatedbecause, unlike music, it is composed ofwords with specific meanings

(B) some languages are inherently more musicaland more suitable to poetic composition thanothers

(C) the translator should be primarily concernedwith reproducing the rhythms and soundpatterns of the original, not with transcribingits meaning exactly

(D) the translator must observe the spirit of theoriginal and also the qualities of expressionthat characterize the language into which theoriginal is translated

(E) poetry is easier to translate if it focuses onphilosophical insights or natural descriptionsrather than on subjective impressions

2. Behind the hope that computers can replace teachersis the idea that the student’s understanding of thesubject being taught consists in knowing facts andrules, the job of a teacher being to make the facts andrules explicit and convey them to the student, eitherby practice drills or by coaching. If that were indeedthe way the mind works, the teacher could transferfacts and rules to the computer, which would replacethe teacher as drillmaster and coach. But sinceunderstanding does not consist merely of knowingfacts and rules, but of the grasp of the generalconcepts underlying them, the hope that thecomputer will eventually replace the teacher isfundamentally misguided.

Which one of the following, if true, would mostseriously undermine the author’s conclusion thatcomputers will not eventually be able to replaceteachers?

(A) Computers are as good as teachers at drillingstudents on facts and rules.

(B) The job of a teacher is to make studentsunderstand the general concepts underlyingspecific facts and rules.

(C) It is possible to program computers so thatthey can teach the understanding of generalconcepts that underlie specific facts and rules.

(D) Because they are not subject to human error,computers are better than teachers atconveying facts and rules.

(E) It is not possible for students to develop anunderstanding of the concepts underlyingfacts and rules through practice drills andcoaching.

GO ON TO THE NEXT PAGE.

44 44SECTION IV

Time—35 minutes

24 Questions

Directions: The questions in this section are based on the reasoning contained in brief statements or passages. For somequestions, more than one of the choices could conceivably answer the question. However, you are to choose the best answer; thatis, the response that most accurately and completely answers the question. You should not make assumptions that are bycommonsense standards implausible, superfluous, or incompatible with the passage. After you have chosen the best answer,blacken the corresponding space on your answer sheet.

Page 25: PT03

4-23-43. If the city council maintains spending at the same

level as this year’s, it can be expected to levy a salestax of 2 percent next year. Thus, if the council levies ahigher tax, it will be because the council is increasingits expenditures.

Which one of the following exhibits a pattern ofreasoning most closely similar to that of theargument above?

(A) If house-building costs are not now rising,builders cannot be expected to increase theprices of houses. Thus, if they decrease theprices of houses, it will be because that actionwill enable them to sell a greater number ofhouses.

(B) If shops wish to reduce shoplifting, theyshould employ more store detectives. Thus, ifshops do not, they will suffer reduced profitsbecause of their losses from stolen goods.

(C) If the companies in the state do not increasetheir workers’ wages this year, the prices theycharge for their goods can be expected to bemuch the same as they were last year. Thus, ifthe companies do increase prices, it will bebecause they have increased wages.

(D) If airlines wish to make profits this year thatare similar to last year’s, they should notincrease their prices this year. Thus, if theycharge more, they should be expected toimprove their services.

(E) If newspaper publishers wish to publish goodpapers, they should employ good journalists.Thus, if they employ poor journalists, it willnot be surprising if their circulation falls as aresult.

4. The mind and the immune system have been shownto be intimately linked, and scientists are consistentlyfinding that doing good deeds benefits one’s immunesystem. The bone marrow and spleen, which producethe white blood cells needed to fight infection, areboth connected by neural pathways to the brain.Recent research has shown that the activity of thesewhite blood cells is stimulated by beneficialchemicals produced by the brain as a result ofmagnanimous behavior.

The statements above, if true, support the view that

(A) good deeds must be based on unselfishmotives

(B) lack of magnanimity is the cause of mostserious illnesses

(C) magnanimous behavior can be regulated bythe presence or absence of certain chemicalsin the brain

(D) magnanimity is beneficial to one’s own interests(E) the number of white blood cells will increase

radically if behavior is consistentlymagnanimous

5. The high cost of production is severely limitingwhich operas are available to the public. These costsnecessitate reliance on large corporate sponsors, whoin return demand that only the most famous operasbe produced. Determining which operas will beproduced should rest only with ticket purchasers atthe box office, not with large corporate sponsors. Ifwe reduce production budgets so that operas can besupported exclusively by box-office receipts anddonations from individuals, then the public will beable to see less famous operas.

Which one of the following, if true, would weakenthe argument?

(A) A few opera ticket purchasers go to the operafor the sake of going to the opera, not to seespecific operatic productions.

(B) The reduction of opera production budgetswould not reduce the desire of large corporatesponsors to support operas.

(C) Without the support of large corporatesponsors, opera companies could not afford toproduce any but the most famous of operas.

(D) Large corporate sponsors will stop supportingopera productions if they are denied controlover which operas will be produced.

(E) The combination of individual donations andbox-office receipts cannot match the amountsof money obtained through sponsorship bylarge corporations.

6. When machines are invented and technologies aredeveloped, they alter the range of choices open to us.The clock, for example, made possible thesynchronization of human affairs, which resulted in anincrease in productivity. At the same time that theclock opened up some avenues, it closed others. It hasbecome harder and harder to live except by the clock,so that now people have no choice in the matter at all.

Which one of the following propositions is bestillustrated by the example presented in the passage?

(A) New machines and technologies can enslave aswell as liberate us.

(B) People should make a concerted effort to freethemselves from the clock.

(C) Some new machines and technologies bring noimprovement to our lives.

(D) The increase in productivity was not worthour dependence on the clock.

(E) Most new machines and technologies makeour lives more synchronized and productive.

GO ON TO THE NEXT PAGE.

44 44

Page 26: PT03

44 -24-

7. To become an expert on a musical instrument, aperson must practice. If people practice a musicalinstrument for three hours each day, they willeventually become experts on that instrument.Therefore, if a person is an expert on a musicalinstrument, that person must have practiced for atleast three hours each day.

Which one of the following most accurately describesa flaw in the reasoning above?

(A) The conclusion fails to take into account thatpeople who practice for three hours every daymight not yet have reached a degree ofproficiency that everyone would considerexpert.

(B) The conclusion fails to take into account thatpracticing for less than three hours each daymay be enough for some people to becomeexperts.

(C) The conclusion fails to take into account that ifa person has not practiced for at least threehours a day, the person has not become anexpert.

(D) The conclusion fails to take into account thatthree consecutive hours of daily practice isnot recommended by all music teachers.

(E) The conclusion fails to take into account thatfew people have the spare time necessary todevote three hours daily to practice.

8. On the basis of incontestable proof that car safety seatswill greatly reduce the number of serious injuriessustained by children in car accidents, laws have beenpassed mandating the use of these seats. Unexpectedly,it has since been found that a large number of childrenwho are riding in safety seats continue to receiveserious injuries that safety seats were specificallydesigned to avoid, and in the prevention of which theyin fact have proven to be effective.

Which one of the following, if true, could by itselfadequately explain the unexpected finding reportedin the passage?

(A) Many parents are defying the law by not usingsafety seats for their children.

(B) Children are more likely to make automobiletrips now than they were before theintroduction of the safety seat.

(C) The high cost of child safety seats has causedmany parents to delay purchasing them.

(D) The car safety seat was not designed to preventall types of injuries, so it is not surprising thatsome injuries are sustained.

(E) The protection afforded by child safety seatsdepends on their being used properly, whichmany parents fail to do.

9. An easy willingness to tell funny stories or jokes aboutoneself is the surest mark of supreme self-confidence.This willingness, often not acquired until late in life, iseven more revealing than is good-natured acquiescencein having others poke fun at one.

Which one of the following inferences is mostsupported by the statements above?

(A) A person who lacks self-confidence will enjoyneither telling nor hearing funny storiesabout himself or herself.

(B) People with high self-confidence do not tellfunny stories or jokes about others.

(C) Highly self-confident people tell funny storiesand jokes in order to let their audience knowthat they are self-confident.

(D) Most people would rather tell a funny story ora joke than listen to one being told.

(E) Telling funny stories or jokes about people intheir presence is a way of expressing one’srespect for them.

GO ON TO THE NEXT PAGE.

44 44

Page 27: PT03

4-25-4Questions 10–11

Nature constantly adjusts the atmospheric carbonlevel. An increase in the level causes the atmosphere tohold more heat, which causes more water to evaporatefrom the oceans, which causes increased rain. Rain washessome carbon from the air into the oceans, where iteventually becomes part of the seabed. A decrease inatmospheric carbon causes the atmosphere to hold lessheat, which causes decreased evaporation from the oceans,which causes less rain, and thus less carbon is washed intothe oceans. Yet some environmentalists worry that burningfossil fuels may raise atmospheric carbon to a dangerouslevel. It is true that a sustained increase would threatenhuman life. But the environmentalists should relax—nature will continually adjust the carbon level.

10. Each of the following can be inferred from theinformation in the passage EXCEPT:

(A) A decrease in the level of atmospheric heatcauses a decrease in the amount of carbonthat rain washes into the oceans from the air.

(B) An increase in the level of carbon in theatmosphere causes increased evaporation ofocean water.

(C) An increase in the level of atmospheric heatcauses increased rainfall.

(D) A decrease in the level of carbon in theatmosphere causes decreased evaporation ofocean water.

(E) A decrease in the level of atmospheric heatcauses a decrease in the level of carbon in theatmosphere.

11. Which one of the following, if true, would mostweaken the argument in the passage?

(A) Plant life cannot survive without atmosphericcarbon.

(B) It is not clear that breathing excess carbon inthe atmosphere will have a negative effect onhuman life.

(C) Carbon is part of the chemical “blanket” thatkeeps the Earth warm enough to sustainhuman life.

(D) Breathing by animals releases almost 30 timesas much carbon as does the burning of fossilfuels.

(E) The natural adjustment process, which occursover millions of years, allows wide fluctuationsin the carbon level in the short term.

12. The more television children watch, the lesscompetent they are in mathematical knowledge.More than a third of children in the United Stateswatch television for more than five hours a day; inSouth Korea the figure is only 7 percent. But whereasless than 15 percent of children in the United Statesunderstand advanced measurement and geometricconcepts, 40 percent of South Korean children arecompetent in these areas. Therefore, if United Stateschildren are to do well in mathematics, they mustwatch less television.

Which one of the following is an assumption uponwhich the argument depends?

(A) Children in the United States are less interestedin advanced measurement and geometricconcepts than are South Korean children.

(B) South Korean children are more disciplinedabout doing schoolwork than are children inthe United States.

(C) Children who want to do well in advancedmeasurement and geometry will watch lesstelevision.

(D) A child’s ability in advanced measurement andgeometry increases if he or she watches lessthan one hour of television a day.

(E) The instruction in advanced measurement andgeometric concepts available to children inthe United States is not substantially worsethan that available to South Korean children.

GO ON TO THE NEXT PAGE.

44 44

Page 28: PT03

44 -26-

Questions 13–14

The only way that bookstores can profitably sell booksat below-market prices is to get the books at a discount frompublishers. Unless bookstores generate a high sales volume,however, they cannot get discounts from publishers. Togenerate such volume, bookstores must either cater to masstastes or have exclusive access to a large specialized market,such as medical textbooks, or both.

13. Which one of the following can be properly inferredfrom the passage?

(A) If a bookstore receives discounts frompublishers, it will profitably sell books atbelow-market prices.

(B) A bookstore that caters to mass tastes or hasexclusive access to a large specialized marketwill have a high sales volume.

(C) A bookstore that profitably sells books atbelow-market prices gets discounts frompublishers.

(D) A bookstore that does not sell books at below-market prices does not get discounts frompublishers.

(E) A bookstore that not only caters to mass tastesbut also has exclusive access to a largespecialized market cannot profitably sellbooks at below-market prices.

14. If all the statements in the passage are true and if it isalso true that a bookstore does not cater to masstastes, which one of the following CANNOT be true?

(A) The bookstore profitably sells some of itsbooks at below-market prices.

(B) The bookstore does not profitably sell any ofits books at below-market prices.

(C) Either the bookstore has exclusive access to alarge specialized market or else it does not geta discount from any publishers.

(D) The bookstore does not have exclusive accessto a large specialized market but profitablysells some of its books at below-marketprices.

(E) The bookstore does not have exclusive accessto a large specialized market, nor does it get adiscount from any publishers.

15. Extinction is the way of nature. Scientists estimate thatover half of the species that have ever come intoexistence on this planet were already extinct beforehumans developed even the most primitive of tools.This constant natural process of species emergence andextinction, however, is ignored by those who wish totrace the blame for more recent extinctions tohumanity’s use of technology, with its consequenteffects on the environment. These people must bemade to understand that the species that have becomeextinct in modern times would have become extinct bynow even if humans had never acquired technology.

Which one of the following identifies a reasoningerror in the passage?

(A) The author mistakenly assumes that technologyhas not caused any harm to the environment.

(B) The author ignores the fact that some speciesthat are not yet extinct are in danger ofextinction.

(C) The author fails to consider that there areprobably species in existence that have not yetbeen identified and studied by scientists.

(D) The author cites scientists who support thetheory that over half of all species that everexisted have become extinct, but fails tomention any scientists who do not supportthat theory.

(E) The author provides no specific evidence thatthe species that have become extinct inmodern times are the same species that wouldhave become extinct in the absence of humantechnology.

GO ON TO THE NEXT PAGE.

44 44

Page 29: PT03

4-27-416. The public is aware of the possibility of biases in the

mass media and distrusts the media as too powerful.The body of information against which the publicevaluates the plausibility of each new media reportcomes, however, from what the public has heard ofthrough the mass media.

If the view above is correct, it provides a reason foraccepting which one of the following conclusions?

(A) If there is a pervasive bias in the presentationof news by the mass media, it would be hardfor the public to discern that bias.

(B) The mass media tailor their reports toconform to a specific political agenda.

(C) The biases that news media impose onreporting tend not to be conscious distortionsbut rather part of a sense they share aboutwhat is interesting and believable.

(D) News reporters and their public hold largelythe same views about what is most importantin society, because news reporters come out ofthat society.

(E) When a news event occurs that contradicts astereotype formerly incorporated intoreporting by the mass media, the public ispredisposed to believe reports of the event.

17. In a bureaucracy, all decisions are arrived at by aprocess that involves many people. There is no oneperson who has the authority to decide whether aproject will proceed or not. As a consequence, inbureaucracies, risky projects are never undertaken.

The conclusion follows logically from the premises ifwhich one of the following is assumed?

(A) All projects in a bureaucracy require risk.(B) Decisive individuals choose not to work in a

bureaucracy.(C) An individual who has decision-making power

will take risks.(D) The only risky projects undertaken are those

for which a single individual hasdecision-making power.

(E) People sometimes take risks as individuals thatthey would not take as part of a group.

18. “Physicalists” expect that ultimately all mentalfunctions will be explainable in neurobiologicalterms. Achieving this goal requires knowledge ofneurons and their basic functions, a knowledge ofhow neurons interact, and a delineation of thepsychological faculties to be explained. At present,there is a substantial amount of fundamentalknowledge about the basic functions of neurons, andthe scope and character of such psychologicalcapacities as visual perception and memory are wellunderstood. Thus, as the physicalists claim, mentalfunctions are bound to receive explanations inneurobiological terms in the near future.

Which one of the following indicates an error in thereasoning in the passage?

(A) The conclusion contradicts the claim of thephysicalists.

(B) The passage fails to describe exactly what iscurrently known about the basic functions ofneurons.

(C) The word “neurobiological” is used as though ithad the same meaning as the word “mental.”

(D) The argument does not indicate whether itwould be useful to explain mental functionsin neurobiological terms.

(E) The passage does not indicate that anyknowledge has been achieved about howneurons interact.

19. Because a large disparity in pay between the public andprivate sectors has developed in recent years, manyexperienced and extremely capable governmentadministrators have quit their posts and takenpositions in private-sector management. Governmentwill be able to recapture these capable administratorsby raising salaries to a level comparable to those of theprivate sector. In that way, the functioning of publicagencies will be improved.

The position taken above presupposes which one ofthe following?

(A) Experience gained from private-sectormanagement will be very valuable ingovernment administration.

(B) The most important factor determining howwell government agencies function is theamount of experience the administrators have.

(C) Unless government action is taken, thedisparity in pay between governmentadministration and private-sectormanagement will continue to increase.

(D) People who moved from jobs in governmentadministration to private-sector managementwould choose to change careers again.

(E) If the disparity in pay between governmentadministration and private-sectormanagement increases, administrators willmove to the public sector in large numbers.

GO ON TO THE NEXT PAGE.

44 44

Page 30: PT03

44 -28-

20. Politician: Homelessness is a serious socialproblem, but further government spending toprovide low-income housing is not the cure forhomelessness. The most cursory glance at thereal-estate section of any major newspaper isenough to show that there is no lack ofhousing units available to rent. So the frequentclaim that people are homeless because of alack of available housing is wrong.

That homelessness is a serious social problem figuresin the argument in which one of the following ways?

(A) It suggests an alternative perspective to the oneadopted in the argument.

(B) It sets out a problem the argument is designedto resolve.

(C) It is compatible either with accepting theconclusion or with denying it.

(D) It summarizes a position the argument as awhole is directed toward discrediting.

(E) It is required in order to establish theconclusion.

21. Leona: If the average consumption of eggs in theUnited States were cut in half, an estimated5,000 lives might be saved each year.

Thomas: How can that be? That would mean that ifpeople adopt this single change in diet for tenyears, the population ten years from now willbe greater by 50,000 people than it otherwisewould have been.

Which one of the following is a statement that Leonacould offer Thomas to clarify her own claim and toaddress the point he has made?

(A) It is possible for the population to grow by5,000 people for every year if the base yearchosen for purposes of comparison is onewith unusually low population growth.

(B) It is accurate to say that 5,000 lives have beensaved as long as 5,000 people who would havedied in a given year as a result of notchanging their diet, did not do so—even ifthey died for some other reason.

(C) If egg consumption were reduced by morethan half, the estimated number of lives savedeach year could be even more than 5,000.

(D) The actual rate of population growth dependsnot only on the birth rate, but also onchanges in life expectancy.

(E) For the average consumption of eggs to be cutby half, many individual consumers wouldhave to cut their own consumption by muchmore than half.

22. The United States Food and Drug Administration(FDA) regulates the introduction of new therapeuticagents into the marketplace. Consequently, it plays acritical role in improving health care in the UnitedStates. While it is those in the academic andgovernment research communities who engage in thelong process of initial discovery and clinical testingof new therapeutic agents, it is the FDA’s role andresponsibility to facilitate the transfer of newdiscoveries from the laboratory to the marketplace.Only after the transfer can important new therapieshelp patients.

Which one of the following statements can beinferred from the passage?

(A) The FDA is responsible for ensuring that anytherapeutic agent that is marketed is thenregulated.

(B) Before new therapeutic agents reach themarketplace they do not help patients.

(C) The research community is responsible for theexcessively long testing period for new drugs,not the FDA.

(D) The FDA should work more closely withresearchers to ensure that the quality oftherapeutic agents is maintained.

(E) If a new medical discovery has beentransferred from the laboratory to themarketplace, it will help patients.

GO ON TO THE NEXT PAGE.

44 44

Page 31: PT03

4-29-423. In a new police program, automobile owners in some

neighborhoods whose cars are not normally drivenbetween 1 A.M. and 5 A.M. can display a special decalin the cars’ windows and authorize police to stop thecars during those hours to check the drivers’ licenses.The theft rate for cars bearing such decals is muchlower than had been usual for cars in thoseneighborhoods.

If it is concluded from the statements above thatautomobile theft has been reduced by the program,which one of the following would it be mostimportant to answer in evaluating that conclusion?

(A) Are owners who are cautious enough to jointhe program taking other special measures toprotect their cars against theft?

(B) In how many neighborhoods is the policeprogram operating?

(C) Are cars in neighborhoods that are activelyparticipating in the program sometimesstolen during daylight hours?

(D) Will owners who have placed decals on theircars’ windows but who find it necessary todrive between 1 A.M. and 5 A.M. be harassedby police?

(E) Are the neighborhoods in which the programhas been put into effect a representative crosssection of neighborhoods with respect to thetypes of automobiles owned by residents?

24. It has been claimed that an action is morally goodonly if it benefits another person and was performedwith that intention; whereas an action that harmsanother person is morally bad either if such harmwas intended or if reasonable forethought wouldhave shown that the action was likely to cause harm.

Which one of the following judgments most closelyconforms to the principle cited above?

(A) Pamela wrote a letter attempting to causetrouble between Edward and his friend; thisaction of Pamela’s was morally bad, eventhough the letter, in fact, had an effect directlyopposite from the one intended.

(B) In order to secure a promotion, Jeffrey devotedhis own time to resolving a backlog ofmedical benefits claims; Jeffrey’s action wasmorally good since it alone enabled Sara’sclaim to be processed in time for her toreceive much-needed treatment.

(C) Intending to help her elderly neighbor byclearing his walkway after a snowstorm,Teresa inadvertently left ice on his steps;because of this exposed ice, her neighbor hada bad fall, thus showing that morally goodactions can have bad consequences.

(D) Marilees, asked by a homeless man for food,gave the man her own sandwich; however,because the man tried to talk while he waseating the sandwich, it caused him to choke,and thus Marilees unintentionally performeda morally bad action.

(E) Jonathan agreed to watch his three-year-oldniece while she played but, becomingengrossed in conversation, did not see her runinto the street where she was hit by a bicycle;even though he intended no harm, Jonathan’saction was morally bad.

44 44

S T O PIF YOU FINISH BEFORE TIME IS CALLED, YOU MAY CHECK YOUR WORK ON THIS SECTION ONLY.

DO NOT WORK ON ANY OTHER SECTION IN THE TEST.

Page 32: PT03

Acknowledgment is made to the following sources from which material has been adapted for use in this test booklet:

From “Economic Goals and Remedies of the AT&T Modified Final Judgment” by Warren G. Lavey and Dennis W. Carlton. TheGoergetown Law Journal, Volume 71, Notes, 1983. © 1983 by the Georgetown Law Journal Association. Used by permission.

Page 33: PT03

4-31-

SIGNATURE / /DATE

LSAT WRITING SAMPLE TOPICSpringfield, faced with a 15 percent decrease in the city budget, must cut services in one area. Write an argument in favor of making the cuts

in one or the other of the following two areas, keeping in mind two guidelines:• Springfield wants the cuts to defer spending in a way that will have minimal impact on the quality of services delivered.• Springfield wants to avoid any negative publicity that could undermine the city government’s reputation for effective management.

One way to cope with the budget cuts is to deny funding for the proposed plan to improve Springfield’s emergency services. This plan wascreated after an article in the local newspaper documented problems caused by outdated equipment and one instance in which slow ambulanceresponse may have been responsible for a person’s death. A new director with a record of successfully improving services in another town wasbrought in to take over the emergency services. She designed a plan that calls for hiring three paramedics specially trained with the mostadvanced equipment. The centerpiece of the plan is the purchase of a computerized dispatching system to improve response time, and a packageto train existing staff to use the system.

The alternative is to make cuts in the educational budget. One cut would deny teachers their requested salary increases for the coming year. Inrecent years, Springfield teacher salaries and benefits have risen to compare with the best in the region. The teacher’s union has publicly statedthat these gains are responsible for the dramatic improvements in student scores on national achievement tests during the last three years. Asecond cut in the education budget would put off construction of an addition to the high school, at a time when serious overcrowding has alreadyforced the placement of four mobile classroom units behind the existing building. Officials predict that two additional units per year will be neededuntil the addition is built.

Page 34: PT03

4 -32-

CONVERSION CHART

For Converting Raw Score to the 120-180 LSAT Scaled ScoreLSAT Prep Test III

REPORTED LOWEST HIGHESTSCORE RAW SCORE RAW SCORE

180 100 101179 99 99178 98 98177 97 97176 96 96175 95 95174 __* __*173 94 94172 93 93171 91 92170 90 90169 89 89168 88 88167 86 87166 85 85165 83 84164 82 82163 80 81162 78 79161 76 77160 75 75159 73 74158 71 72157 69 70156 67 68155 65 66154 63 64153 61 62152 60 60151 58 59150 56 57149 54 55148 52 53147 50 51146 48 49145 47 47144 45 46143 43 44142 41 42141 40 40140 38 39139 36 37138 35 35137 33 34136 32 32135 30 31134 29 29133 28 28132 26 27131 25 25130 24 24129 23 23128 22 22127 21 21126 20 20125 19 19124 18 18123 17 17122 16 16121 __* __*120 0 15

DIRECTIONS:

1. Use the Answer Key on the next page to check youranswers.

2. Use the Scoring Worksheet below to compute yourRaw Score.

3. Use the Score Conversion Chart to convert your Raw Score into the 120-180 scale.

SCORING WORKSHEET

1. Enter the number of questions you answeredcorrectly in each section

NUMBERCORRECT

SECTION I. . . . . . . . . . .

SECTION II . . . . . . . . . .

SECTION III . . . . . . . . .

SECTION IV . . . . . . . . .

2. Enter the sum here: THIS IS YOURRAW SCORE.

*There is no raw score that will produce this scaled score for this test.

Page 35: PT03

4-33-

1. D2. B3. A4. E5. C6. A7. D

1. C2. E3. D4. B5. E6. C7. A

1. E2. D3. A4. C5. D6. C7. C

1. D2. C3. C4. D5. C6. A7. B

8. C9. A10. C11. A12. A13. D14. A

8. B9. D10. E11. D12. D13. A14. B

8. C9. E10. C11. D12. A13. D14. A

8. E9. A10. E11. E12. E13. C14. D

15. D16. D17. E18. D19. A20. B21. C

15. D16. C17. C18. C19. B20. C21. B

15. D16. E17. E18. D19. B20. A21. E

15. E16. A17. D18. E19. D20. C21. B

22. D23. B24. C

22. A23. C24. A25. E

22. A23. C24. D25. D26. C27. B28. B

22. B23. A24. E

SECTION I

SECTION II

SECTION III

SECTION IV

Page 36: PT03

*LSAT is a registered trademark of the Law School Admission Council.

1-800-KAP-TEST | kaptest.com

ÖLL3072AzäLL3072A

Printed in USA ©2008 Kaplan, Inc.